0% found this document useful (0 votes)
2 views

depsSln_1

The document provides solutions to various Delta Epsilon questions, including tolerable errors in cube volume, limit verification using formal definitions, and proving the uniqueness of limits. It also discusses the Squeeze Theorem and the non-existence of certain limits. Each solution is methodically derived with mathematical reasoning and proofs.

Uploaded by

Kamal Elamraoui
Copyright
© © All Rights Reserved
Available Formats
Download as PDF, TXT or read online on Scribd
0% found this document useful (0 votes)
2 views

depsSln_1

The document provides solutions to various Delta Epsilon questions, including tolerable errors in cube volume, limit verification using formal definitions, and proving the uniqueness of limits. It also discusses the Squeeze Theorem and the non-existence of certain limits. Each solution is methodically derived with mathematical reasoning and proofs.

Uploaded by

Kamal Elamraoui
Copyright
© © All Rights Reserved
Available Formats
Download as PDF, TXT or read online on Scribd
You are on page 1/ 3

Solutions to More Delta Epsilon Questions

1) What is the largest tolerable error in the length of the edge of a cube if the volume of
the cube must be within ±1.2% of 8, 000 cm3 .
Solution. Let the length of the edge of the cube be x. The volume of the cube must be
1.2
within 100 8000 = 96 of 8000. Hence
√ √
7904 = 8000 − 96 ≤ x3 ≤ 8000 + 96 = 8096 ⇐⇒
3 3
7904 ≤ x ≤ 8096

To have x3 = 8000, we need x = 20. The maximum allowed error in x is the smaller of

3
√ √
8096 − 20 and 20 − 3 7904, which is 3 8096 − 20 ≈ 0.0797 .

2) Use the formal definition of the limit to verify that

lim x−22 =0
x→2 1+x

Solution. We must prove that

x−2
∀ ε > 0 ∃ δ > 0 such that 1+x2 < ε whenever 0 < |x − 2| < δ

Let ε > 0.
Choose δ =“to be filled in later”.
Then, whenever 0 < |x − 2| < δ, we have
x−2
1+x2 ≤ |x − 2| since 1 + x2 ≥ 1

≤ε if δ ≤ ε
x−2
We have now shown that, whenever 0 < |x − 2| < δ, we have 1+x2
< ε, provided δ
obeys δ ≤ ε. So we may choose δ = ε. That is, we may choose “to be filled in later”
above to be ε.

3) Use the formal definition of the limit to verify that



lim 2x + 3 = 3
x→3

Solution. We must prove that



∀ ε > 0 ∃ δ > 0 such that 2x + 3 − 3 < ε whenever 0 < |x − 3| < δ

1
Let ε > 0.
Choose δ =“to be filled in later”.
Then, whenever 0 < |x − 3| < δ, we have
√ 2
2x + 3 − 3 = (2x+3)−3

2x+3+3
= √ 2x−6
2x+3+3
2(x−3)

≤ 3 since 2x + 3 ≥ 0
< 32 δ
≤ε if δ ≤ 23 ε

We have now shown that, whenever 0 < |x − 3| < δ, we have 2x + 3 − 3 < ε, provided
3 3
δ obeys δ ≤ 2 ε. So we may choose δ = 2 ε. That is, we may choose “to be filled in later”
3
above to be 2
ε.

4) Prove that lim sin x1 does not exist. Do so by directly verifying that the formal definition
x→0
1
of lim sin x
=L cannot be satisfied for any L.
x→0
Solution. Let L ∈ IR. Assume that lim sin x1 = L. We will derive a contradiction. To
x→0
do so, we must show that the statement

∀ ε > 0 ∃ δ > 0 such that sin x1 − L < ε whenever 0 < |x| < δ

is false.
To do so, it suffices for us to exhibit one “bad” value of ε. Choose ε = 10−6 . (This is
just a guess, which happens to work out. Lots of other guesses would work too.) Now
that we have chosen a value of ε, we must show that the statement

∃ δ > 0 such that sin x1 − L < 10−6 whenever 0 < |x| < δ

is false. To do this, we must show that, for every δ > 0, the statement

sin x1 − L < 10−6 whenever 0 < |x| < δ (∗)

is false.
So let δ > 0. If the statement (∗) were true, this would imply that sin x1 takes values only
between L − 10−6 and L + 10−6 for all 0 < |x| < δ. Note that L − 10−6 ≤ y ≤ L + 10−6 is
an interval of length 2 × 10−6 . As x runs over the interval (0, δ), (so that, in particular,
0 < |x| < δ) x1 covers the set δ1 , ∞ . This contains many intervals of length 2π and


hence many periods of sin. So, as x runs over the interval (0, δ), sin x1 covers all of [−1, 1].
1
But [−1, 1] is an interval of length 2 and so cannot be contained in L − 2 ≤ y ≤ L + 21 .
So (∗) is false.

2
5) Prove that limits are unique. That is, prove that if lim f (x) = L and lim f (x) = M ,
x→a x→a
then L = M .
Solution. We are told that,

∀ ε > 0 ∃ δ1 > 0 such that f (x) − L < ε whenever 0 < |x − a| < δ1

and

∀ ε > 0 ∃ δ2 > 0 such that f (x) − M < ε whenever 0 < |x − a| < δ2

and we are to conclude that L = M . Consider any ε > 0. There is a δ1 > 0 such that
f (x) − L < ε for all 0 < |x − a| < δ1 and a δ2 > 0 such that 0 < f (x) − M < ε
for all |x − a| < δ2 . (These two δ’s are allowed to be different. That’s why I have
chosen two different names, δ1 and δ2 .) Pick any x0 obeying both 0 < |x0 − a| < δ1 and
0 < |x0 − a| < δ2 . Then

|L − M | = L − f (x0 ) + f (x0 ) − M ≤ L − f (x0 ) + f (x0 ) − M < 2ε.

We have just shown that |L − M | < 2ε for every ε > 0. This forces L = M . (If L 6= M ,
we may choose ε = |L−M4
|
and conclude that |L − M | < 2 |L−M
4
|
, which would imply that
|L−M |
2
< 0, which is impossible.)

6) Suppose that a < b < c, that f (x) < g(x) < h(x) for all a < x < c, and that lim f (x) =
x→b
lim h(x) = L. Prove that lim g(x) = L. (This is called the Squeeze Theorem.)
x→b x→b
Solution. We must prove that

∀ ε > 0 ∃ δ > 0 such that g(x) − L < ε whenever 0 < |x − b| < δ

Let ε > 0. Note that we are told that there is a δ1 > 0 such that f (x) − L < ε for all
0 < |x − b| < δ1 and a δ2 > 0 such that h(x) − L < ε for all 0 < |x − b| < δ2 .
Choose δ =“to be filled in later”.
Then, whenever 0 < |x − b| < δ, we have
(1) (2) (2) (3)
L − ε < f (x) < g(x) < h(x) < L + ε
=⇒ L − ε < g(x) < L + ε
=⇒ g(x) − L < ε
provided δ ≤ δ1 (for the inequality (1)) and δ ≤ c − b, b − a (so that a < x < c for the
inequalities (2)) and δ ≤ δ2 (for the inequality (3)). We have now shown that, whenever
0 < |x − b| < δ, we have g(x) − L < ε, provided δ obeys δ ≤ δ1 and δ ≤ δ2 and δ ≤ c − b
and δ ≤ b − a. So we may choose δ = min{δ1 , δ2 , b − a, c − b}. That is, we may choose
“to be filled in later” above to be min{δ1 , δ2 , b − a, c − b}.

You might also like